Jump to content

Alex Andrews

Members
  • Posts

    6
  • Joined

  • Last visited

Profile Information

  • Location
    UK

Alex Andrews's Achievements

Lepton

Lepton (1/13)

1

Reputation

  1. OK, thanks very much, I will definitely have a watch. I can understand how such an explanation would predict the oberved result, but it still just seems an inordinately complex one hence my citing of Occam's Razor. Also, when you say that it is not possible to guarantee which slit the electron would pass through, what if you were to build a "corridor" from the electron source to one of the slits so that the electron could only pass through the one slit, would you still get the observed interference pattern? And why is this effect only observable for small objects (as I understand it)? Surely size shouldn't be a factor if an object is being interfered with by a copy of itself? Sorry - so many questions!
  2. OK, well I don't pretend to understand even half of what you have said, but I am happy to accept that you know what you're talking about! I'll see if I can get my head around some of it at least. Then maybe I have misunderstood because I thought that de Broglie's pilot wave was in essence the wave part of wave-particle duality. In retrospect, I think it was a mistake to bring up wave-particle duality as now I don't think it's really relevant. I guess what I'm trying to say is that the observed results of the double slit experiment (with single electrons) do not appear (to me) to be consistent with the electron passing through both slits but do appear consistent with a wave front passing through both slits and as a consequence causing the path of the electron to deviate from its original course (Occam's Razor springs to mind). What exactly that wave is I have no idea but I suggested the possibility of a bow wave as a real-world analogy. Out of interest, suppose the experiment is conducted with slits A and B and the electrons are fired through slit B. If slit A is covered up, presumably the electrons pass straight through without any sort of deviation? And if slit B is instead covered up, do electrons still "go through" slit A? Thanks, Alex
  3. Because the wave is not intrinsically associated with the particle, it is created by the motion of the particle. It is completely distinct from the particle itself. And it is completely beyond my ken to test it which is why I have come here to discuss it.
  4. OK, thanks I shall have a read and try to get my head around that. So, I've had a quick read up on pilot waves and inelastic scattering. I might have misunderstood pilot waves, but the theory seems to be that a particle intrinsically has an associated wave: So, my understanding of this is that particles have an associated wave which I guess goes hand-in-hand with wave-particle duality. That is not what I am suggesting. My proposal is that a moving particle creates some sort of bow-wave: in my analogy, a moving boat creates a bow-wave in the water in front of it. I am not suggesting that the boat itself is some sort of wave or that it has some sort of wave intrinsically associated with it; it is its motion through the water which creates the bow-wave in the water and similarly the electron moving through space. With regards to inelastic scattering: Interaction between the bow-wave and the particle (or in my analogy, the bow-wave and the boat) could easily influence the speed of the particle (boat) to some greater or lesser degree when the bow-wave itself is interfered with by other waves. Have I misunderstood? Alex
  5. Hi, Thanks all for your replies. As you can probably tell, my knowledge in this area is fairly superficial. OK, so the reason I wanted to check my understanding is as follows: If, as is observed, single electrons fired through a double-slit produce an intereference pattern, it does not follow logically to infer that the electron passes through both slits and somehow intereferes with itself. I believe that when detectors are placed at the slits (a) the electron is detected passing through only one of the slits, and (b) the interference pattern is dampened down. However, the resultant interference pattern cannot be disputed, so what has caused it? Would it not make more sense (both instinctively and logically) if it were the case that while the electron passed through one of the slits, a wave also passed through both slits? The resultant two wave fronts would then interfere with each other as you would expect and in turn the electron, causing the electron's path to deviate as observed. After all, (again as I understand it!), it is only the path of the electron that suffers the "interference", not its speed or energy level or any other characteristic. I liken this to a boat travelling on water: as it moves forward it produces a bow-wave. Something similar could happen with electrons (or any particle) moving through space, whereby it produces some sort of bow-wave which I guess we currently can't detect. If a moving particle has some sort of associated bow-wave, then wouldn't that also chime rather well with the wave-particle duality principle? It would certainly explain why the electron is detected passing through only one slit, and possibly why the presence of the detectors dampens the result (the detectors absorbs some of the wave energy). This explanation makes much more sense to me, but then I'm far from an expert in this area and maybe this idea has been considered and dismissed previously. Any thoughts? Alex
  6. I would like to ask a question about (Eggert's?) Double Slit Experiment, but first I would like to check my understanding of the current interpretation. My understanding is that when electrons are fired individually at a double slit, the observed result is an inteference pattern similar to that observed when a wave passes through a double-slit. The conclusion inferred from this observed pattern is that the electron is actually in two places simultaneously (ie it passes through both slits at the same time) and consequently interferes with itself to produce the observed pattern. This is then used as the basis for Quantum Theory. Is my understanding correct? Thanks very much, Alex
×
×
  • Create New...

Important Information

We have placed cookies on your device to help make this website better. You can adjust your cookie settings, otherwise we'll assume you're okay to continue.